11
$\begingroup$

Suppose $A$ and $B$ are finitely generated Abelian groups. Are all exact sequences of the form $0 \rightarrow A \rightarrow A \oplus B \rightarrow B \rightarrow 0$ split?

If not, is there an example?

$\endgroup$
7
  • 2
    $\begingroup$ What's wrong with the canonical inclusion $B \to A \oplus B$ ? $\endgroup$ May 20, 2014 at 15:44
  • 9
    $\begingroup$ It isn't stated what the maps are, so it is not clear that that would give a splitting. $\endgroup$
    – R.P.
    May 20, 2014 at 15:58
  • $\begingroup$ @Alberto: that $0\to N\to G\to Q\to 0$ splits means that the map $Q\to G$ lifts the projection $G\to Q$. This can fail with the canonical inclusion. $\endgroup$
    – YCor
    May 20, 2014 at 16:07
  • 7
    $\begingroup$ See Steven Landsburg's answer to the following question: mathoverflow.net/questions/157938/… $\endgroup$ May 20, 2014 at 16:14
  • $\begingroup$ Obviously in the the question homomorphisms are arbitrary, not the canonical ones. The answer is "yes, they're all split" by Landsburg's answer, see the link given by Dag Oskar Madsen. $\endgroup$
    – YCor
    May 20, 2014 at 16:37

2 Answers 2

26
$\begingroup$

This is true more generally for finitely generated modules over a noetherian ring. Your question is equivalent to asking whether the sequence $$0\rightarrow \operatorname{Hom}(B,A)\rightarrow \operatorname{Hom}(A\oplus B,A)\rightarrow \operatorname{Hom}(A,A)$$ is surjective on the right. To prove this, it suffices to localize and then complete at an arbitrary prime $P$, so we can assume we're working over a complete local ring where $P$ is the maximal ideal. Now it suffices to check surjectivity after modding out an arbitrary power $P^n$, which allows us to assume that all the modules are of finite length. Surjectivity follows because the lengths of the left-hand and right-hand modules add up to the length of the module in the middle.

Edited to add: The comments above (which I read after I posted this) remind me that I've posted this same argument before. If people think this instance should be deleted, I'm fine with that.

$\endgroup$
6
$\begingroup$

I thought it worth adding a reference to this:

Miyata, Takehiko, Note on direct summands of modules, J. Math. Kyoto Univ. 7 1967 65–69.

In the paper, the question of the OP is attributed to Matsumura and the solution to Toda. Then a short argument is given for this case.

The author generalizes this result to the following (which is a quote from MathSciNet):

"Let $R$ be a commutative, noetherian ring and let $A$ be an $R$-algebra of finite type. Moreover, let $M$ be a finitely generated $A$-module and let $N$ be a submodule of $M$. Using the usual tools of homological algebra and noetherian ring theory, the author establishes the following pair of results.

Theorem 1: If $M$ is isomorphic to $N\oplus M/N$, then $N$ is a direct summand of $M$.

Theorem 2: If $0\to N\otimes_R T\to M\otimes_R T$ is exact for all $A$-modules $T$ (i.e., $N$ is pure), then $N$ is a direct summand of $M$."

$\endgroup$
2
  • $\begingroup$ Note: I think "$R$-algebra of finite type" means "finitely generated $R$-algebra". $\endgroup$
    – YCor
    Jul 16, 2019 at 19:23
  • $\begingroup$ As does Wikipedia. $\endgroup$ Jul 17, 2019 at 7:44

Your Answer

By clicking “Post Your Answer”, you agree to our terms of service and acknowledge you have read our privacy policy.

Not the answer you're looking for? Browse other questions tagged or ask your own question.